Những câu hỏi liên quan
Thu Phương Nguyễn
Xem chi tiết
ミ★Zero ❄ ( Hoàng Nhật )
25 tháng 4 2021 lúc 9:45

\(\frac{x^2}{y+1}+\frac{y+1}{4}\ge x;\frac{y^2}{z+1}+\frac{z+1}{4}\ge y;\frac{z^2}{x+1}+\frac{x+1}{4}\ge z\)

\(\Rightarrow VT\ge\frac{3}{4}\left(x+y+z\right)-\frac{3}{4}\ge\frac{3}{4}.2=\frac{3}{2}\)

Bình luận (0)
 Khách vãng lai đã xóa
Lê Trường Lân
Xem chi tiết
Lê Trường Lân
15 tháng 5 2020 lúc 17:04

Bài 3 thì \(\le1\)

Bài 4 thì \(\ge\frac{3}{4}\) nhé

Bình luận (0)
 Khách vãng lai đã xóa
ONLINE SWORD ART
Xem chi tiết
Lâm ngọc mai
Xem chi tiết
Kudo Shinichi
1 tháng 1 2020 lúc 10:22

Áp dụng BĐT Cauchy - Schwarz ta có :

\(VT=\frac{x}{\sqrt[3]{yz}}+\frac{y}{\sqrt[3]{xz}}+\frac{z}{\sqrt[3]{xy}}=\frac{x^2}{\sqrt[3]{x^3yz}}+\frac{y^2}{\sqrt[3]{y^3xz}}+\frac{z^2}{\sqrt[3]{z^3xy}}\)

\(\ge\frac{\left(x+y+z\right)^2}{\sqrt[3]{x^3yz}+\sqrt[3]{y^3xz}+\sqrt[3]{z^3xy}}\left(1\right)\)

Áp dụng BĐT : AM - GM :

\(\sqrt[3]{x^3yz}\le\frac{x^2+xyz+1}{3};\sqrt[3]{y^3xz}\le\frac{y^2+xyz+1}{3};\sqrt[3]{z^3xy}\le\frac{z^2+xyz+1}{3}\)

\(\Rightarrow\sqrt[3]{x^3yz}+\sqrt[3]{y^3xz}+\sqrt[3]{z^3xy}\le\frac{x^2+y^2+z^2+3xyz+3}{3}=2+xyz\)

Theo BĐT AM - GM :

\(x^2+y^2+z^2\ge3\sqrt[3]{x^2y^2z^2}\Leftrightarrow3\sqrt[3]{x^2y^2z^2}\le3\Leftrightarrow xyz\le1\)

Do đó : \(\sqrt[3]{x^3yz}+\sqrt[3]{y^3xz}+\sqrt[3]{z^3xy}\le3\left(2\right)\)

Tư (1) , (2) và sử dụng hệ quả :
\(x^2+y^2+z^2\ge xy+yz+zx:\)

\(\Rightarrow VT\ge\frac{\left(x+y+z\right)^2}{3}=\frac{x^2+y^2+z^2+2\left(xy+yz+xz\right)}{3}\ge\frac{3\left(xy+yz+xz\right)}{3}\)\(=xy+yz+xz\)

Ta có đpcm 

Dấu " = " xảy ra khi \(x=y=z=1\)

Chúc bạn học tốt !!!

Bình luận (0)
 Khách vãng lai đã xóa
nhóm54
Xem chi tiết
Ngọc Vĩ
Xem chi tiết
Hà Ngọc Khánh
17 tháng 6 2016 lúc 16:49

http://diendantoanhoc.net/topic/160455-%C4%91%E1%BB%81-to%C3%A1n-v%C3%B2ng-2-tuy%E1%BB%83n-sinh-10-chuy%C3%AAn-b%C3%ACnh-thu%E1%BA%ADn-2016-2017/

Bình luận (0)
Đặng Minh Triều
16 tháng 6 2016 lúc 22:25

bài của tui mà -_-

Bình luận (0)
Ngọc Vĩ
16 tháng 6 2016 lúc 22:30

hihi k biết làm nên đăng ^^

Bình luận (0)
Nguyễn Hữu Quang
Xem chi tiết
Nguyễn Thị Thương Hoài
9 tháng 7 2023 lúc 17:11

Bài 3:

a, (\(x\)+y+z)2

=((\(x\)+y) +z)2

= (\(x\) + y)2 + 2(\(x\) + y)z + z2

\(x^2\) + 2\(xy\) + y2 + 2\(xz\) + 2yz + z2

=\(x^2\) + y2 + z2 + 2\(xy\) + 2\(xz\) + 2yz

 

Bình luận (0)
Nguyễn Thị Thương Hoài
9 tháng 7 2023 lúc 17:14

b, (\(x-y\))(\(x^2\) + y2 + z2 - \(xy\) - yz - \(xz\))

\(x^3\) + \(xy^2\) + \(xz^2\) - \(x^2\)y - \(xyz\) - \(x^2\)z - y3 

Đến dây ta thấy xuất hiện \(x^3\) - y3 khác với đề bài, em xem lại đề bài nhé

Bình luận (0)
Nguyễn Thị Thương Hoài
9 tháng 7 2023 lúc 17:26

c,

(\(x\) + y + z)3 

=(\(x\) + y)3 + 3(\(x\) + y)2z + 3(\(x\)+y)z2 + z3

\(x^3\) + 3\(x^2\)y + 3\(xy^{2^{ }}\) + y3 +  3(\(x\)+y)z(\(x\) + y + z) + z3

\(x^3\) + y3 + z3 + 3\(xy\)(\(x\) + y) + 3(\(x+y\))z(\(x+y+z\))

\(x^3\) + y3 + z+ 3(\(x\) + y)( \(xy\) + z\(x\) + yz + z2)

\(x^3\) + y3 + z3 + 3(\(x\) + y){(\(xy+xz\)) + (yz + z2)}

\(x^3\) + y3 + z3 + 3(\(x\) + y){ \(x\)( y +z) + z(y+z)}

\(x^3\) + y3 + z3 + 3(\(x\) + y)(y+z)(\(x+z\)) (đpcm)

 

 

Bình luận (0)
Phạm Tiến Minh
Xem chi tiết
Xem chi tiết
Akai Haruma
26 tháng 12 2017 lúc 10:02

Lời giải:

Áp dụng BĐT Cauchy-Schwarz ta có:

\(\text{VT}=\frac{x}{\sqrt[3]{yz}}+\frac{y}{\sqrt[3]{xz}}+\frac{z}{\sqrt[3]{xy}}=\frac{x^2}{\sqrt[3]{x^3yz}}+\frac{y^2}{\sqrt[3]{y^3xz}}+\frac{z^2}{\sqrt[3]{z^3xy}}\)

\(\geq \frac{(x+y+z)^2}{\sqrt[3]{x^3yz}+\sqrt[3]{y^3xz}+\sqrt[3]{z^3xy}}\) (1)

Áp dụng BĐT Am-Gm:

\(\sqrt[3]{x^3yz}\leq \frac{x^2+xyz+1}{3}; \sqrt[3]{y^3xz}\leq \frac{y^2+xyz+1}{3}; \sqrt[3]{z^3xy}\leq \frac{z^2+xyz+1}{3}\)

\(\Rightarrow \sqrt[3]{x^3yz}+\sqrt[3]{y^3xz}+\sqrt[3]{z^3xy}\leq \frac{x^2+y^2+z^2+3xyz+3}{3}=2+xyz\)

Theo BĐT AM-GM:

\(x^2+y^2+z^2\geq 3\sqrt[3]{x^2y^2z^2}\Leftrightarrow 3\sqrt[3]{x^2y^2z^2}\leq 3\Leftrightarrow xyz\leq 1\)

Do đó: \(\sqrt[3]{x^3yz}+\sqrt[3]{y^3xz}+\sqrt[3]{z^3xy}\leq 3\) (2)

Từ (1),(2) và sử dụng hệ quả \(x^2+y^2+z^2\geq xy+yz+xz\) :

\(\Rightarrow \text{VT}\geq \frac{(x+y+z)^2}{3}=\frac{x^2+y^2+z^2+2(xy+yz+xz)}{3}\geq \frac{3(xy+yz+xz)}{3}=xy+yz+xz\)

Ta có đpcm

Dấu bằng xảy ra khi \(x=y=z=1\)

Bình luận (0)
Lightning Farron
27 tháng 12 2017 lúc 17:50

Áp dụng BĐT AM-GM ta có:

\(VT\ge\dfrac{x}{\dfrac{y+z+1}{3}}+\dfrac{y}{\dfrac{x+z+1}{3}}+\dfrac{z}{\dfrac{x+y+1}{3}}\)

Cần chứng minh \(\dfrac{9x}{y+z+1}+\dfrac{9y}{x+z+1}+\dfrac{9z}{x+y+1}\ge3\left(xy+yz+xz\right)\)

Cauchy-Schwarz: \(VT=\dfrac{9x^2}{xy+xz+x}+\dfrac{9y^2}{xy+yz+y}+\dfrac{9z^2}{xz+yz+z}\)

\(\ge\dfrac{9\left(x+y+z\right)^2}{2\left(xy+yz+xz\right)+x+y+z}\ge\left(x+y+z\right)^2\)

BĐT cuối đúng vì dễ thấy: \(\left(x+y+z\right)^2\ge3\left(xy+yz+xz\right)\)

Bình luận (0)